Multiplying two function operators

In summary: DateTime>.In summary, the conversation discusses the expansion of ##F(\hat A)## and gives an example of ##e^{a\hat A}##. It also mentions that if ##[\hat A, \hat B] \neq 0##, then ##e^{\hat A} e^{\hat B} \neq e^{\hat A + \hat B}##. The individual steps to obtain the specific formula are explained, and it is also shown that the formula only holds if certain conditions are met.
  • #1
Pencilvester
184
42
TL;DR Summary
I’m having trouble working out how ##e^{\hat A} e^{\hat B} = e^{\hat A + \hat B} e^{[\hat A , ~ \hat B ]/2}##
I am reading Zettili’s “Quantum Mechanics: Concepts and Applications” and I am in the section on functions of operators. It starts with how ##F(\hat A)## can be Taylor expanded and gives the particular and familiar example: $$e^{a \hat A} = \sum_{n=0}^\infty \frac{a^n}{n!} \hat A^n \tag{2.109}$$ Later it says how if ##[\hat A , ~ \hat B] \neq 0## then ##e^{\hat A} e^{\hat B} \neq e^{\hat A + \hat B}##. Then here’s where I am having trouble: It says, “using (2.109) we can ascertain that $$e^{\hat A} e^{\hat B} = e^{\hat A + \hat B} e^{[\hat A , ~ \hat B ]/2},\\
e^{\hat A} \hat B e^{- \hat A} = \hat B + [\hat A , ~ \hat B] + \frac{1}{2!} [\hat A , ~ [\hat A , ~ \hat B ]] + \frac{1}{3!} [\hat A , ~ [\hat A , ~ [\hat A , ~ \hat B ]]] + \cdots”$$ I guess I am not a savvy enough mathematician to figure out how they got either of these from eq. 2.109. Can someone help me out?
 
Physics news on Phys.org
  • #2
It isn't very funny to do, but at least the first couple of terms can be calculated: Just use distribution and multiply the sums:
$$
\left( 1+A +\dfrac{1}{2}A^2+\dfrac{1}{6}A^3 +\ldots \right)\cdot \left( 1+B +\dfrac{1}{2}B^2+\dfrac{1}{6}B^3 +\ldots \right) = \ldots
$$
and compare it to the right hand side. Just do not forget that matrix multiplication isn't commutative, so ##AB## is different from ##BA## and e.g. ##(A+B)^2= A^2+AB+BA+B^2##.
 
  • #4
Pencilvester said:
Summary: I’m having trouble working out how ##e^{\hat A} e^{\hat B} = e^{\hat A + \hat B} e^{[\hat A , ~ \hat B ]/2}##

I am reading Zettili’s “Quantum Mechanics: Concepts and Applications” and I am in the section on functions of operators. It starts with how ##F(\hat A)## can be Taylor expanded and gives the particular and familiar example: $$e^{a \hat A} = \sum_{n=0}^\infty \frac{a^n}{n!} \hat A^n \tag{2.109}$$ Later it says how if ##[\hat A , ~ \hat B] \neq 0## then ##e^{\hat A} e^{\hat B} \neq e^{\hat A + \hat B}##. Then here’s where I am having trouble: It says, “using (2.109) we can ascertain that $$e^{\hat A} e^{\hat B} = e^{\hat A + \hat B} e^{[\hat A , ~ \hat B ]/2},\\
e^{\hat A} \hat B e^{- \hat A} = \hat B + [\hat A , ~ \hat B] + \frac{1}{2!} [\hat A , ~ [\hat A , ~ \hat B ]] + \frac{1}{3!} [\hat A , ~ [\hat A , ~ [\hat A , ~ \hat B ]]] + \cdots”$$ I guess I am not a savvy enough mathematician to figure out how they got either of these from eq. 2.109. Can someone help me out?
Using (2.109) you get
$$\mathrm{d}_a \exp(a \hat{A})=\hat{A} \exp(a \hat{A})=\exp(a \hat{A}) \hat{A}.$$
Now define
$$\hat{F}(a)=\exp(a \hat{A}) \hat{B} \exp(-a \hat{A}).$$
We (formally) expand ##\hat{F}## in a power series
$$\hat{F}(a)=\hat{F}(0)+a \hat{F}'(0) +\frac{a^2}{2} \hat{F}''(0)+...$$
We have ##\hat{F}(0)=\hat{B}##,
$$\hat{F}'(a)=\exp(a \hat{A}) [\hat{A},\hat{B}] \exp(-a \hat{A}).$$
Now use this formula again, but instead of ##\hat{B}## for ##[\hat{A},\hat{B}]##, which leads to
$$\hat{F}''(a)=\exp(a \hat{A})[\hat{A}, [\hat{A},\hat{B}]] \exp(-a \hat{A}).$$
It's easy to see that the ##j##-th derivative is
$$\hat{F}^{(j)}(a)=\exp(a \hat{A}) [\hat{A},\hat{B}]_j \exp(-a \hat{A}),$$
where
$$[\hat{A},\hat{B}]_{j}=[\underbrace{\hat{A},[\hat{A},\ldots}_{j \quad \text{times}},B].$$
Setting ##a=0## in the power series, you get the 2nd formula you have trouble with.

The 2nd formula only holds if
$$[\hat{A},[\hat{A},\hat{B}]]=[\hat{B},[\hat{A},\hat{B}]]=0, \qquad (*)$$
To show this, we define
$$\hat{F}(a)=\exp[a(\hat{A}+\hat{B})].$$
The same series expansion technique together with the assumption (*) yields
$$\hat{F}(a) \hat{A} \hat{F}^{-1}(a)=\hat{A} + a [\hat{A}+\hat{B},\hat{A}]=\hat{A}+a [\hat{B},\hat{A}].$$
From this we get
$$\hat{F}(a) \hat{A}=\hat{A} \hat{F}(a) - a [\hat{A},\hat{B}]\hat{F}(a).$$
On the other hand with this we have
$$\mathrm{d}_a \hat{F}(a) = \hat{F}(a)(\hat{A}+\hat{B}) =\hat{F}(a) \hat{B} + \hat{A} \hat{F}(a)-a [\hat{A},\hat{B}]\hat{F}(a).$$
Taking into account again (*) it's easy to show that this differential equation is solved by
$$\hat{F}(a)=\exp(a \hat{A}) \exp(a \hat{B}) \exp \left (-\frac{a^2}{2} [\hat{A},\hat{B}] \right).$$
Setting ##a=1## gives
$$\hat{F}(1)=\exp(\hat{A}+\hat{B}) = \exp(\hat{A}) \exp(\hat{B}) \exp \left (-\frac{1}{2} [\hat{A},\hat{B}] \right).$$
Again, it's important to keep in mind that this holds only, if (*) is fulfilled. Nevertheless the formula is of some use in applications.
 
  • Like
Likes Pencilvester and dextercioby
  • #5
fresh_42 said:
It isn't very funny to do, but at least the first couple of terms can be calculated: Just use distribution and multiply the sums:
$$
\left( 1+A +\dfrac{1}{2}A^2+\dfrac{1}{6}A^3 +\ldots \right)\cdot \left( 1+B +\dfrac{1}{2}B^2+\dfrac{1}{6}B^3 +\ldots \right) = \ldots
$$
and compare it to the right hand side.
You’re right, this is not at all very fun to do, but I began doing it, and I’m running into a problem. If you start to expand ##e^{A+B}e^{[A, ~ B]/2}## you get $$(1+A+B+ \cdots + \frac{1}{6} A^2B + \cdots )(1+ \frac{1}{2} AB + \cdots )$$Which gives us ##\frac{2}{3} A^2B + \cdots## but the expansion of ##e^A e^B## says we should only have ##\frac{1}{2} A^2B##. What am I screwing up?
 
  • #6
I don't know what happens with the additional condition that ##[A,[A,B]]=[B,[B,A]]=0## has to hold, see post #4.

You can find the complete CBH formula here where the other twelfth can be found:
https://www.physicsforums.com/insights/lie-algebras-a-walkthrough-the-representations/
or if you don't like the proof in post #4, maybe you like this one (also with the additional conditions on commutativity of the higher order terms):
https://www.physicsforums.com/threads/commutator-identity.966227/#post-6133835
 
  • Like
Likes vanhees71
  • #7
vanhees71 said:
The 2nd formula only holds if
$$[\hat{A},[\hat{A},\hat{B}]]=[\hat{B},[\hat{A},\hat{B}]]=0, \qquad (*)$$
...
Taking into account again (*) it's easy to show that this differential equation is solved by
$$\hat{F}(a)=\exp(a \hat{A}) \exp(a \hat{B}) \exp \left (-\frac{a^2}{2} [\hat{A},\hat{B}] \right).$$
Wow. Okay, if Zettili intended his readers to work all that out just from eq. 2.109, then my math skills must be far from where they should be to study this book. Thanks for the explanation though!
 
  • #8
Hm, I don't think one gets this without knowing this (or perhaps another) simpler trick :-)).
 
  • Like
Likes Pencilvester
  • #9
vanhees71 said:
The 2nd formula only holds if
[^A,[^A,^B]]=[^B,[^A,^B]]=0,(∗)​
Is this premise valid for the use of this special form of the BCH in QFT when constructing field operators expanding in terms creation and annihilation operators?
 
  • #10
In which context do you need it? Sometimes it's also sufficient to just use the series for the general case up to the first few terms (e.g., in the derivation of the path-integral expression for the generating functional for Green's functions etc.).
 
  • #11
vanhees71 said:
In which context do you need it? Sometimes it's also sufficient to just use the series for the general case up to the first few terms (e.g., in the derivation of the path-integral expression for the generating functional for Green's functions etc.).
In the context of field operator construction with annihilation and creation operators. The conmutator of these(-I) commutes with either one (creation or annihilation) operator so it seems to obey (*) in your post and in the same was is done in the free fields construction.
 
  • Like
Likes vanhees71

Related to Multiplying two function operators

1. What is the definition of multiplying two function operators?

Multiplying two function operators is a mathematical operation that combines two functions to create a new function. It is denoted by using the multiplication symbol (*) between the two functions.

2. How does multiplying two function operators work?

Multiplying two function operators involves applying the first function to the input, and then applying the second function to the output of the first function. This creates a new function that represents the combination of the two original functions.

3. What are the properties of multiplying two function operators?

The properties of multiplying two function operators include commutativity, associativity, and distributivity. This means that the order in which the functions are multiplied does not matter, and the result will be the same regardless. Additionally, the operation can be distributed over addition or subtraction of functions.

4. How is multiplying two function operators used in real life?

Multiplying two function operators is commonly used in physics, engineering, and other scientific fields to represent complex systems or processes. It can also be used in economics and finance to model relationships between variables.

5. Are there any limitations to multiplying two function operators?

Yes, there are limitations to multiplying two function operators. One limitation is that both functions must be defined for the same input values. Additionally, the result of multiplying two function operators may not always be a function, depending on the functions being multiplied.

Similar threads

Replies
2
Views
143
Replies
4
Views
1K
Replies
16
Views
1K
Replies
14
Views
1K
  • Quantum Physics
Replies
1
Views
700
  • Quantum Physics
Replies
1
Views
669
  • Quantum Physics
2
Replies
56
Views
3K
  • Quantum Physics
Replies
4
Views
2K
Replies
1
Views
734
  • Quantum Physics
Replies
2
Views
990
Back
Top